واسمحوا veca = <- 2،3> و vecb = <- 5، k>. ابحث عن k بحيث يكون veca و vecb متعامدين. ابحث عن k بحيث تكون a و b متعامدة؟

واسمحوا veca = <- 2،3> و vecb = <- 5، k>. ابحث عن k بحيث يكون veca و vecb متعامدين. ابحث عن k بحيث تكون a و b متعامدة؟
Anonim

إجابة:

# "استذكر ذلك ، لاثنين من المتجهات:" qquad vec {a} ، vec {b} qquad "لدينا:" #

# qquad vec {a} quad "و" quad vec {b} qquad quad "متعامدات" qquad qquad hArr qquad qquad vec {a} cdot vec {b} = 0. #

# "هكذا:" #

# qquad <-2، 3> quad "و" quad <-5، k> qquad quad "متعامد" qquad qquad hArr #

# qquad qquad <-2، 3> cdot <-5، k> = 0 qquad qquad hArr #

# qquad qquad qquad (-2) (-5) + (3) (k) = 0 qquad qquad hArr #

# qquad qquad qquad qquad qquad qquad 10 + 3 k = 0 qquad qquad hArr #

# qquad qquad qquad qquad qquad quad 3 k = -10 qquad qquad hArr #

# qquad qquad qquad qquad qquad qquad quad k = -10/3. #

# "لذا ، من البداية إلى النهاية هنا:" #

# qquad <-2، 3> quad "و" quad <-5، k> qquad quad "متعامد" qquad qquad hArr #

# qquad qquad qquad qquad qquad qquad qquad qquad qquad k = -10/3. #

# "وهكذا ، نستنتج:" #

# qquad qquad qquad qquad qquad qquad qquad qquad qquad k = -10/3. #